LSAT and Law School Admissions Forum

Get expert LSAT preparation and law school admissions advice from PowerScore Test Preparation.

 brcibake
  • Posts: 55
  • Joined: Jul 19, 2017
|
#39552
I was trying to decide between A and B but ultimately choose B. They both seem correct to me. What makes A stronger than B?
Thank you
 Eric Ockert
PowerScore Staff
  • PowerScore Staff
  • Posts: 164
  • Joined: Sep 28, 2011
|
#39637
Answer choice (A) is discussed by both authors here. Author A mentions "serious reductions in waterfowl" in Line 11 while author B mentions the canvasback as "endangered by purple loosestrife" in Lines 53-55. These would prove that the two authors agree that "some" populations tend to decrease.

Author A mentions the "disastrous effect" of purple loosestrife in Lines 5-6. So it is clear that author A would agree with answer choice (B). However, author B never really mentions any actual disastrous effect. Without a doubt, author B would agree that others believe its effects are disastrous. However, at no point does author B commit to any kind of agreement with that belief.

Get the most out of your LSAT Prep Plus subscription.

Analyze and track your performance with our Testing and Analytics Package.